Abstract Algebra ch13 Dummit Foote2 PDF

You might also like

Download as pdf or txt
Download as pdf or txt
You are on page 1of 19

Abstract Algebra

Chapter 13 - Field Theory


David S. Dummit & Richard M. Foote
Solutions by positrn0802
positron0802@mail.com

Contents
13 Field Theory 1
13.1 Basic Theory and Field Extensions . . . . . . . . . . . . . . . . . . . . . . . . . . . . . . 1
13.2 Algebraic Extensions . . . . . . . . . . . . . . . . . . . . . . . . . . . . . . . . . . . . . . 4
13.3 Classical Straightedge and Compass Constructions . . . . . . . . . . . . . . . . . . . . . 9
13.4 Splitting Fields and Algebraic Closures . . . . . . . . . . . . . . . . . . . . . . . . . . . . 11
13.5 Separable and Inseparable Extension . . . . . . . . . . . . . . . . . . . . . . . . . . . . . 12
13.6 Cyclotomic Polynomials and Extensions . . . . . . . . . . . . . . . . . . . . . . . . . . . 15

13 Field Theory
13.1 Basic Theory and Field Extensions
Exercise 13.1.1.
p(x) = x3 + 9x + 6 is irreducible in Z[x] by Eisenstein Criterion with p = 3. By Gauss Lemma, then it
is irreducible in Q[x]. To find (1 + )1 , we apply the Euclidean algorithm (long division) to p(x) and
1 + x. We find
x3 + 9x + 6 = (1 + x)(x2 x + 10) 4.
Evaluating at , we find (1 + )(2 + 10) = 4. Therefore

2 + 10
(1 + )1 = .
4

Exercise 13.1.2.
Let f (x) = x3 2x 2. f is irreducible over Z by Eisenstein Criterion with p = 2, hence over Q by
Gauss Lemma. Now, if is a root of f , then 3 = 2 + 2. Hence

(1 + )(1 + + 2 ) = 1 + 2 + 22 + 3 = 3 + 4 + 22 .
1+
For computing , first we compute (1+ +2 )1 . Applying the Euclidean algorithm, we obtain
1 + + 2
x3 2x 2 = (x2 + x + 1)(x 1) 2x 1,

and
x2 x 7 9
x3 2x 2 = (2x + 1)( ) .
2 4 8 8
Evaluating at , from this equalities we obtain
8 2 7
(2 + + 1)( 1) = 2 + 1 and (2 + 1)1 = ( ).
9 2 4 8
Combining these two equations we obtain
8 2 2 7
( + + 1)( 1)( ) = 1.
9 2 4 8

1
13.1 Basic Theory and Field Extensions

So,
8 2 7 22 5
(2 + + 1)1 = ( 1)( ) = + + ,
9 2 4 8 3 3 3
where we used 3 = 2 + 2 again. Therefore,

1+ 22 5 2 2 1
2
= (1 + )( + + ) = + + .
1++ 3 3 3 3 3 3

Exercise 13.1.3.
Since 03 + 0 + 1 = 1 and 11 + 1 + 1 = 1 in F2 , then x3 + x + 1 is irreducible over F2 . Since is root of
x3 + x + 1, then 3 = 1 = + 1. Hence, the powers of in F2 () are

, 2 , 3 = + 1, 4 = 2 + , 5 = 2 + + 1, 6 = 2 + 1, and 7 = 1.

Exercise 13.1.4.
Denote this map by . Then

(a + b 2 + c + d 2) = a + c b 2 d 2 = (a + b 2) + (c + d 2),

and
((a + b 2) (c + d 2)) = (ac + 2bd + (ad + bc) 2)

= ac + 2bd (ad + bc) 2

= (a b 2)(c d 2)

= (a + b 2)(c + d 2),

hence
is an homomorphism. Moreover, if (a+b 2) = (c+d 2), then
ab 2 = cd 2, hence (since

2 6 Q) a = b and c = d, so is injective. Also, given a+ b 2 Q( 2), then (a b 2) = a + b 2,
so is surjective. Therefore, is an isomorphism of Q( 2) with itself.
Exercise 13.1.5.
Let = p/q be a root of a monic polynomial p(x) = xn + + a1 x + a0 over Z, with gcd(p, q) = 1.
Then
p p p
( )n + an1 ( )n1 + + a1 + a0 = 0.
q q q
Multiplying this equation by q n one obtains

pn + an1 pn1 q + + a1 pq n1 + a0 q n = 0
q(an1 pn1 + + a1 pq n2 + a0 q n1 ) = pn .

Thus, every prime that divides q divides pn as well, so divides p. Since gcd(p, q) = 1, there is no prime
dividing q, hence q = 1. The result follows.
Exercise 13.1.6.
This is straightforward. If
an n + an1 n1 + + a1 + a0 = 0,
then
(an )n + an1 (an )n1 + an an2 (an )n2 + + an2
n a1 (an ) + an1
n a0
= ann n + an1
n an1 n1 + an1
n an2 n2 + + an1
n a1 + an1
n a0
= an1
n (an n + an1 n1 + an2 n2 + + a1 + a0 ) = 0.

Exercise 13.1.7.
If x3 nx + 2 is reducible it must have a linear factor, hence a root. By the Rational Root Theorem,
if is a root of x3 nx + 2, then must divide its constant term, so the possibilities are = 1, 2.
If = 1 or 2, then n = 3; if = 1, then n = 1; and if = 2, then n = 5. Therefore, x3 nx + 2 is
irreducible for n 6= 1, 3, 5.

2
13.1 Basic Theory and Field Extensions

Exercise 13.1.8.
We subdivide this exercise in cases and subcases.
If x5 ax 1 is reducible then it has a root (linear factor) or is a product of two irreducible
polynomials of degrees 2 and 3.
Case 1. If x5 ax 1 has a root, then, by the Rational Root Theorem, it must be = 1. If = 1
is a root, then a = 0. If = 1 is a root, then a = 2.
Case 2. Now, suppose that there exists f (x) and g(x) irreducible monic polynomials over Z of
degrees 2 and 3 respectively, such that x5 ax 1 = f (x)g(x). Write f (x) = x2 + bx + c and
g(x) = x3 + rx2 + sx + t, where b, c, r, s, t Z. Then

x5 ax 1 = (x2 + bx + c)(x3 + rx2 + sx + t)


= x5 + (b + r)x4 + (br + c + s)x3 + (bs + cr + t)x2 + (bt + cs) + tc.

Equating coefficients leads to


b+r =0
br + c + s = 0
bs + cr + t = 0
bt + cs = a
ct = 1.
From ct = 1 we deduce (c, t) = (1, 1) of (c, t) = (1, 1), which give us two cases.
Case 2.1. First suppose (c, t) = (1, 1). Then the system of equations reduces to

b+r =0
br 1 + s = 0
bs r + 1 = 0
b s = a.

Now, put b = r into second and third equations to obtain r2 1 + s = 0 and rs r + 1 = 0, that
is, r2 + 1 s = 0 and rs + r 1 = 0. Adding these last two equations we obtain r2 + rs + r s = 0.
Thus r2 + rs + r + s = 2s, so (r + 1)(r + s) = 2s. Now, from r2 + 1 s = 0 we have r2 = s 1, so then
r2 + rs + r s = 0 becomes rs + r = 1, that is, r(s + 1) = 1. Hence, r = 1 and s = 0, or r = 1 and
s = 2. If r = 1 and s = 0, then (r + 1)(r + s) = 2s leads to 2 = 0, a contradiction. If r = 1 and
s = 2, it leads to 0 = 4, another contradiction.
Therefore, (c, t) = (1, 1) is impossible. We now pass to the case (c, t) = (1, 1).
Case 2.2. Suppose that (c, t) = (1, 1). The system of equations reduces to

b+r =0
br + 1 + s = 0
bs + r 1 = 0
b + s = a.

Adding the second and third equation we obtain b(r + s) + r + s = 0, so that (b + 1)(r + s) = 0. Then
b = 1 or r = s, so one more time we have two cases. If r = s, then br + 1 + s = 0 becomes
br + 1 r = 0. Hence, b = r and br + 1 r = 0 gives r2 + r 1 = 0. By the Rational Root Theorem,
this equation has no roots on Z. Since r Z, we have a contradiction. Now suppose b = 1. From
b = r we obtain r = 1, so, from br + 1 + s = 0 we obtain s = 0. Finally, from b + s = a we
obtain a = 1. Therefore, the solution (b, c, r, s, t) = (1, 1, 1, 0, 1) is consistent and we obtain the
factorization

x5 ax 1 = (x2 + bx + c)(x3 + rx2 + sx + t) = (x2 x + 1)(x3 + x2 1).

3
13.2 Algebraic Extensions

13.2 Algebraic Extensions


Exercise 13.2.1.
Since the characteristic of F is p, its prime subfield is (isomorphic to) Fp = Z/pZ. We can consider F as
a vector space over Fp . Since F is finite, then [F : Fp ] = n for some n Z+ . Therefore

|F| = |Fp |[F:Fp ] = pn .

Exercise 13.2.2.
Note that g and h are irreducible over F2 and F3 . Now, is is a root of g, then F2 () = F2 /(g(x))
has 4 elements and F3 ()
= F3 /(g(x)) has 9 elements. Furthermore, is 2 is a root of h, then F2 (2 )
=
F2 /(h(x)) has 8 elements and F3 (2 )
= F3 /(h(x)) has 27 elements.
The multiplication table for F2 /(g(x)) is

0 1 x x+1
0 0 0 0 0
1 0 1 x x+1
x 0 x x+1 x
x+1 0 x+1 x x

The multiplication table for F3 /(g(x)) is

0 1 2 x x+1 x+2 2x 2x + 1 2x + 2
0 0 0 0 0 0 0 0 0 0
1 0 1 2 x x+1 x+2 2x 2x + 1 2x + 2
2 0 2 1 2x 2x + 2 2x + 1 x x+2 x+1
x 0 x 2x 2x + 1 1 x+1 x+2 2x + 2 2
x+1 0 x+1 2x + 2 1 x+2 2x 2 x 2x + 1
x+2 0 x+2 2x + 1 x+1 2x 2 2x + 2 1 x
2x 0 2x x x+2 2 2x + 2 2x + 1 x+1 1
2x + 1 0 2x + 1 x+2 2x + 2 x 1 x+1 2 2x
2x + 2 0 2x + 2 x+1 2 2x + 1 x 1 2x x+2

In both cases, x is a generator of the cyclic group of nonzero elements.


Exercise 13.2.3.
Since 1 + i 6 Q, its minimal polynomial is of degree at least 2. We try conjugation, and obtain

(x (1 + i))(x (1 i)) = x2 2x + 2,

which is irreducible by Eisenstein with p = 2. Therefore, the minimal polynomial is x2 2x + 2.


Exercise 13.2.4.

First, note that (2+ 3)2 = 4+4 3+3 = 7+4 3. Let = 2+ 3. Then 2 4 = 7+4 384 3 = 1,
hence is a root of x2 4x+1. Moreover,
x2 4x+1 isirreducible over Q (because 6 Q), so x2 4x+1
is the minimal polynomial
of 2 + 3. Therefore, 2 + 3 has degree 2 over Q.
Now, let = 3 2 and = 1 + + 2 . Then Q(), so Q Q() Q(). We have [Q() : Q] =
[Q() : Q()][Q() : Q]. Note that [Q() : Q] = 3 since has minimal polynomial x3 2 over Q, so
[Q() : Q] = 1 or 3. For a contradiction, suppose [Q() : Q] = 1, that is, Q() = Q so Q. Then

2 = (1 + + 2 )2 = 1 + 2 + 32 + 23 + 4 = 5 + 4 + 32 ,

where we used 3 = 2. So

2 3 = 5 + 2 + 32 3(1 + + 2 ) = 2 ,

hence = 2 + 3 + 2 Q() = Q, a contradiction. Therefore, [Q() : Q] = 3.

4
13.2 Algebraic Extensions

Exercise 13.2.5.
Since the polynomials have degree 3, if they were reducible they must have a linear factor, hence a root

3 3
in F . Note
that every element of F is of the form a + bi, where a, b Q. The roots of x 2 are 2, 32
and 23 2, where is the primitive 3rd root of unity, i.e., = exp(2i/3) = cos(2/3) + i sin(2/3) =

12 + 23 . Since 3 6 Q, none of this elements is in F , hence x3 2 is irreducible over F . Similarly, the

roots of x3 3 are 3 3, 3 3 and 2 3 3, and by the same argument non of this elements is in F . Hence
x3 3 is irreducible over F .
Exercise 13.2.6.
We have to prove that F (1 , . . . , n ) is the smallest field containing F (1 ), . . . , F (n ). Clearly F (i )
F (1 , . . . , n ) for all 1 i n. Now let K be a field such that F (i ) K for all i. If is an
element of F (1 , . . . , n ), then is of the form = a1 1 + + an n , where a1 , . . . , an F . Every
ai i is in K, hence K. Thus F (1 , . . . , n ) K. Therefore, F (1 , . . . , n ) contains all F (i )
and is contained in every field containing all F (i ), hence F (1 , . . . , n ) is the composite of the fields
F (1 ), F (2 ), . . . , F (n ).
Exercise 13.2.7.

Since 2 + 3 is inQ( 2, 3), Q( 2 + 3) Q( 2, 3). For the other direction we have to
clearly
prove that 2 and 2 are in Q( 2 + 3). Let = 2 + 3. Then

2 = 5 + 2 6, 3 = 11 2 + 9 3 and 4 = 37 + 15 6.

So
1 3 1
( 9),
2= 3 = (11 3 ) and 4 = 49 + 20 6.
2 2

Therefore
2 Q() and
3 Q(), so Q( 2, 3) Q( 2 + 3). The equality follows. Hence,
[Q( 2 + 3) : Q] = [Q( 2, 3) : Q] = 4.
We also have

4 102 = (49 + 20 6) 10(5 + 2 6) = 1, so 4 102 + 1 = 0.

Since [Q( 2 + 3) : Q] = 4, then x4 10x2 + 1 is irreducible over Q, and is satisfied by 2 + 3.
Exercise 13.2.8.

The elements of F ( D1 , D2 ) can be written in the form
p p p
a + b D1 + c D2 + d D1 D2 , where a, b, c, d F.

We have p p p p p p
[F ( D1 , D2 ) : F ] = [F ( D1 , D2 ) : F ( D1 )][F ( D1 ) : F ].

Since [F ( D1 ) :
F ] = 2, then
[F ( D1 , D2 ) : F ] can be 2 or 4. Now, 2[F ( D1 , D2 ) : F ] = 2 if
and
only if [F ( D1 , D2 ): F ( D1 )] = 1, and that occurs exactly when x D2 is reducible in F ( D1 )
(i.e., when D2 F ( D1 )), that is, if there exists a, b F such that
p p
(a + b D1 )2 = D2 , so that a2 + 2ab D1 + b2 D12 = D2 .

Note that ab = 0 as ab 6= 0 implies D1 F , contrary to the hypothesis. Then a = 0 or b = 0. If
2
b = 0, then D2 is a square in F , contrary to the hypothesis. If a = 0, then b D1 = D2 , and thus
D2 2 2
D1 D2 = ( b ) , so D1 D2 is a square in F . So, x D2 is reducible in F ( D1 ) if and only if D1 D2 is a
square in F . The result follows.
Exercise
p 13.2.9.

Suppose a + b = m + n for some m, n F ,p then a + b = m + n + 2 mn. Since b is not a

square in F , this means b = 2 mn. We also have a + b n = m, so

q
b = 2 n( a + b n).

5
13.2 Algebraic Extensions

Hence,
q
b = 2 n(a + b) 2n

( b + 2n)2 = 4n(a + b)

b + 4n b + 4n2 = 4n(a + b)
b + 4n2 4na = 0

4a 16a2 16b
n=
8
p 2n
a2 b = .
a

Therefore, since a and n are in F , a2 b is in F .
Now suppose that a2 b is a square in F , so that a2 b F . We prove that there exists m, n F
p
such that a + b = m + n. Let

a + a2 b a a2 b
m= and n = .
2 2
p
Note that m and n are in F as char(F ) 6= 2. We claim a + b = m + n. Indeed, we have
p p
(a + b) + 2 a2 b + (a b) a+ b+ a b 2
m= =( ) ,
4 2
and p p
(a + b) 2 a2 b + (a b) a+ b a b 2
n= =( ) .
4 2
Thus p p p p
a+ b+ a b a+ b a b
m= and n= .
2 2
Therefore, p p p p

q
a+ b+ a b a+ b a b
m+ n= + = a+ b,
2 2
as claimed. p
Now, we this to determine when the field
p Q( a + b), a, b Q, is biquadratic over Q. If a2 b

is apsquare in Q and b is not, we have Q( a + b) = Q( m + n) = Q( m, n), so by last exercise

Q( a + b) is biquadratic over Q when a2 b is a square in Q, and neither b, m, n or mn are squares
in Q. Since
a + a2 b a a2 b b
mn = = ,
2 2 4
p
then mn is never a square when b isnt. Thus, Q( a + b) is biquadratic over Q exactly when a2 b
is a square in Q and neither b, m nor n is a square in Q.
Exercise
p 13.2.10.
p 2
Note that 3 + 2 2 = 3 + 8. Recalling last exercise with a = 3 and b = 8, we have p a b

= 98 = 1

pand b
is a square in Q = 8 is not. Hence, we find (m = 2 and n = 1 from p

last exercise) 3 + 8 = 2+1.

Therefore, Q( 3 + 2 2) = Q( 2) and the degree of the extension Q( 3 + 2 2) over Q is 2.
Exercise 13.2.11.
(a) First, note that the conjugation map a + bi a bi is an isomorphism of C, so it takes squares roots
to square roots, and maps numbers of the first quadrant to the fourth (and reciprocally). Since 3 + 4i
its conjugate is the square of root of 3 4i in the fourth
of 3 + 4i in thefirst quadrant,
is the square root
quadrant, so is 3 4i. Hencep 3 + 4i and 3 4i are conjugates each other. Now, we use Exercise

9 again. Note that 3 + 4i = 3 + 16. With a = 3 and b = 16, wehave a2 b =25 is a square
in Q and b = 16 is not. Hence, we find m = 1 and n = 4and thus 3 + 4i = 1 + 4 = 1 + 2i.
Furthermore, we find 3 4i = 1 2i. Therefore, 3 + 4i + 3 4i = 4, i.e., 3 + 4i + 3 4i Q.

6
13.2 Algebraic Extensions

p p
(b) Let = 1 + 3 + 1 3. Then

q q
2 = ( 1 + 3 + 1 3)2 = (1 + 3) + (2 1 + 3) + (1 3) = 6.

Since x2 6 is irreducible over Q (Eisenstein p = 2), then has degree 2 over Q.


Exercise 13.2.12.
Let E be a subfield of K containing F . Then

[K : F ] = [K : E][E : F ] = p.

Since p is prime, either [K : E] = 1 or [E : F ] = 1. The result follows.


Exercise 13.2.13.
m
Note that, for all 1 k n, we have [Q(1 , . . . ,
k ) : Q(1 , . . . , k1
)] = 1 or 2. Then [F : Q] = 2
for some m N. Suppose 3 2 F . Then Q Q( 3 2) F , so [Q( 3 2) : Q] divides [F : Q], that is, 3
divides 2m , a contradiction. Hence, 3 2 6 F .
Exercise 13.2.14.
Since 2 F (), clearly F (2 ) F (). Thus we have to prove F (2 ). For this purpose, consider
the polynomial p(x) = x2 2 , so that p() = 0. Note that F (2 ) if and only if p(x) is reducible
in F (2 ). For a contradiction, suppose p(x) is irreducible in F (2 ), so that [F () : F (2 )] = 2. Thus

[F () : F ] = [F () : F (2 )][F (2 ) : F ] = 2[F (2 ) : F ],

so [F () : F ] is even, a contradiction. Therefore, p(x) is reducible in F (2 ) and F (2 ).


Exercise 13.2.15.
We follow the hint. Suppose there exists a counterexample. Let be of minimal degree such that F ()
is not formally real and having minimal polynomial f of odd degree, say deg f = 2k +1 for some k N.
Since F () is not formally real, then 1 can be express as a sum of squares in F ()
= F [x]/((f (x))).
Then, the exists polynomials p1 (x), . . . , pm (x), g(x) such that

1 + f (x)g(x) = (p1 (x))2 + + (pm (x))2 .

As every element in F [x]/((f (x)) can be written as a polynomial in with degree less than deg f ,
we have deg pi < 2k + 1 for all i. Thus, the degree in the right hand of the equation is less than
4k + 1, so deg g < 2k + 1 as well. We prove that the degree of g is odd by proving that the degree of
(p1 (x))2 + + (pm (x))2 is even, because then the equation 1 + f (x)g(x) = (p1 (x))2 + + (pm (x))2
implies the result. Let d be the maximal degree over all pi , we prove that x2d is the leading term of
(p1 (x))2 + + (pm (x))2 . Note that x2d is a sum of squares (of the leading coefficients of the pi s of
maximal degree). Now, since F is formally real, 0 cant be expressed as a sum of squares in F . Indeed,
Pl Pl1
if i=1 a2i = 0, then i=1 (ai /al )2 = 1. Therefore x2d 6= 0, so the degree of (p1 (x))2 + + (pm (x))2
is 2d, as claimed. Hence, the degree of g must be odd by the assertion above. Then g must contain an
irreducible factor of odd degree, say h(x). Since deg g < deg f , we have deg h < deg f as well. Let be
a root of h(x), hence a root of g(x). Then

f (x)g(x)
1 + h(x) = (p1 (x))2 + + (pm (x))2 ,
h(x)

so 1 is a square in F [x]/((h(x))
= F (), which means F () is not formally real. Therefore, is a root
of an odd degree polynomial h such that F () is not formally real. Since deg h < deg f , this contradicts
the minimality of . The result follows.
Exercise 13.2.16.
Let r R be nonzero. Since r is algebraic over F , there exist an irreducible polynomial p(x) =
a0 + a1 x + + xn F [x] such that p(r) = 0. Note that a0 6= 0 since p is irreducible. Then
r1 = a10 (r
n1
+ + a1 ). Since ai F R and r R, we have r1 R.

7
13.2 Algebraic Extensions

Exercise 13.2.17.
Let p(x) be an irreducible factor of f (g(x)) of degree m. Let be a root of p(x). Since p is irreducible,
then [F () : F ] = deg p(x) = m. Now, since p(x) divides f (g(x)), we have f (g()) = 0 and thus g()
is a root of f (x). Since f is irreducible, this means n = [F (g()) : F ]. Note that F (g()) F ().
Therefore,
m = [F () : F ] = [F () : F (g())][F (g()) : F ] = [F () : F (g())] n,
so n divides m, that is, deg f divides deg p.
Exercise 13.2.18.
(a) We follow the hint. Since k[t] is an UFD and k(t) is its field of fractions, then, by Gauss Lemma,
P (X)tQ(X) is irreducible in k((t))[X] is and only if it is irreducible in (k[t])[X]. Note that (k[t])[X] =
(k[X])[t]. Since P (X) tQ(X) is linear in (k[X])[t], is clearly irreducible in (k[X])[t] (i.e., in (k[t])[X]),
hence in (k(t))[X]. Thus, P (X) tQ(X) is irreducible in k(t). Now, x is clearly a root of P (X) tQ(X)
P (x)
since P (x) tQ(x) = P (x) Q(x) = P (x) P (x) = 0.
Q(x)
(b) Let n = max{degP (x), degQ(x)}. Write

P (x) = an xn + + a1 x + a0 and Q(x) = bn xn + + b1 x + b0 ,

where ai , bi k for all i, so at least one of an or bn is nonzero. The degree of P (X) tQ(X) is clearly
n, we prove is n. If an or bn is zero then clearly deg (P (X) tQ(X)) = n. Suppose an , bn 6= 0. Then
an , bn k, but t 6 k (as t k(x)), it cannot be that an = tbn . Thus (an tbn )X n 6= 0, so the degree of
P (X) tQ(X) is n.
(c) Since P (X) tQ(X) is irreducible over k(t) and x is a root by part (a), then [k(x) : k(t)] =
degP (X) tQ(X), and this degree equals max{degP (x), degQ(x)} by part (b).
Exercise 13.2.19.
(a) Fix in K. Since K is (in particular) a commutative ring, we have (a + b) = a + b and
(a) = (a) for all a, b, K. If, in particular, F , we have the result.
(b) Fix a basis for K as a vector space over F . By part (a), for every K we can associate a
F -linear transformation T . Denote by T the matrix of T with respect to the basis fixed above.
Then define : K Mn (F ) by () = T . We claim is an isomorphism. Indeed, if , K,
then T(+) (k) = ( + )(k) = k + k = T (k) + T (k) for every k K, hence T(+) = T + T .
We also have T() (k) = ()(k) = kk = T (k)T (k) for every k K, so T() = T T . Thus
( + ) = () + () and () = ()() (since the basis is fixed), so is an homomorphism.
Now, if () = (), then k = k for every k K, so letting k = 1 we find that is injective.
Therefore, (K) is isomorphic to a subfield of Mn (F ), so the ring Mn (F ) contains an isomorphic copy
of every extension of F of degree n.
Exercise 13.2.20.
The characteristic polynomial of A is p(x) = det(Ix A). For every k K, we have (I A)k =
k Ak = k k = 0, so det(I A) = 0 in K. Therefore,
p() = 0.
3 3 3
Now, consider
the field Q(
2) with basis
{1, 2, 4} over
Q. Denote the elements of this basis by
e1 = 1, e2 = 3 2 and e3 = 3 4. Let = 3 2 and = 1 + 3 2 + 3 4. Then (e1 ) = e2 , (e2 ) = e3 and
(e3 ) = 2e1 . We also have (e1 ) = e1 + e2 + e3 , (e2 ) = 2e1 + e2 + e3 and (e3 ) = 2e1 + 2e2 + e3 . Thus,
the associated matrices of the their linear transformations are, respectively,

0 0 2 1 2 2
A = 1 0 0 and A = 1 1 2 .
0 1 0 1 1 1

The characteristic
polynomial of A is x3 2, hence is the monic polynomial of degree 3 satisfied by
3
= 2. Furthermore, the characteristic polynomial
of A is x3 3x2 3x 1, hence is the monic
3 3
polynomial of degree 3 satisfied by = 1 + 2 + 4.

8
13.3 Classical Straightedge and Compass Constructions

Exercise 13.2.21.
The matrix of the linear transformation "multiplication by " on K is found by acting 
of in the
 basis
a bD
1, D. We have (1) = = a + b D and ( D) = a D + bD. Hence the matrix is . Now
b a

 
a bD
let : K M2 (Q) be defined by (a + b D) = .
b a
We have

     
a + c (b + d)D a bD c dD
(a + b D + c + d D) = = + = (a + b D) + (c + d D),
b+d a+c b a d c
and

     
ac + bdD (ad + bc)D a
bD c dD
((a + b D) (c + d D)) = =
ad + bc ac + bdD b
a d c

= (a + b D)(c + d D),
so is an homomorphism. Since K is a field, its ideals are {0} and K, so ker() is trivial or K. Since
(K) is clearly non-zero, then ker() 6= K and thus ker() = {0}. Hence, is injective. Therefore,
is an isomorphism of K with a subfield of M2 (Q).
Exercise 13.2.22.
Define : K1 K2 K1 K2 by (a, b) = ab. We prove that is F -bilinear. Let a, a1 , a2 K and
b, b1 , b2 K2 . Then
((a1 , b) + (a2 , b)) = (a1 + a2 , b) = (a1 + a2 )b = a1 b + a2 b = (a1 , b) + (a2 , b),
and
((a,1 b) + (a, b2 )) = (a, b1 + b2 ) = a(b1 + b2 ) = ab1 + ab1 = (a, b1 ) + (a, b2 ).
We also have, for r F , (ar, b) = (ar)b = a(rb) = (rb). Therefore, is a F -bilinear map. Hence,
induces a F -algebra homomorphism : K1 F K2 K1 K2 . We use to prove both directions. Note
that K1 F K2 have dimension [K1 : F ][K2 : F ] as a vector space over F .
First, we suppose [K1 K2 : F ] = [K1 : F ][K2 : F ] and prove K1 F K2 is a field. In this case
K1 F K2 and K1 K2 have the same dimension over F . Let L = (K1 F K2 ). We claim L = K1 K2 ,
i.e. is surjective. Note that L contains K1 and K2 . Since L is a subring of K1 K2 containing K1 (or
K2 ), then L is a field (Exercise 16). Hence, L is a field containing both K1 and K2 . Since K1 K2 is
the smallest such field (by definition), we have L = K1 K2 . Therefore is surjective, as claimed. So,
is an F -algebra surjective homomorphism between F -algebras of the same dimension, hence is an
isomorphism. Thus, K1 F K2 is a field.
Now suppose that K1 F K2 is a field. In this case is a field homomorphism. Therefore,
is either injective or trivial. It is clearly nontrivial since (1 1) = 1, so it is injective. Hence,
[K1 : F ][K2 : F ] [K1 K2 : F ]. As we already have [K1 K2 : F ] [K1 : F ][K2 : F ] (Proposition 21 of
the book), the equality follows.

13.3 Classical Straightedge and Compass Constructions


Exercise 13.3.1.
Suppose the 9-gon is constructible. It has angles of 40 . Since we can bisect an angle by straightedge and
compass, the angle of 20 would be constructible. But then cos 20 and sin 20 would be constructible
too, a contradiction (see proof of Theorem 24).
Exercise 13.3.2.
Let O, P, Q and R be the points marked in the figure below.

9
13.3 Classical Straightedge and Compass Constructions

Then = QP O, = RQO, = QRO, and is an exterior angle of 4P RO. Since 4P QO is


isosceles, then = QP O = QOP . Since is an exterior angle of 4P QO, it equals the sum of the
two remote interior angles, i.e., equals QP O + QOP . This two angles equals , hence = 2. Now,
since 4QRO is isosceles, then = . Finally, since is an exterior angle of 4P RO, equals the sum of
the two remote interior angles, which are and . Therefore, = + = + = 3.
Exercise 13.3.3.
We follow the hint. The distances a, b, x, y and x k are marked in the figure below.

From the figure, using similar triangles for (a), (b) and (c), and Pythagoras Theorem for (d), the 4
relations are clear. Hence, we have

1 k2 b+k y 1 k2
y= , x=a , = and (1 k 2 ) + (b + k)2 = (1 + a)2 .
1+a 1+a xk 3k
3ky
So, 1 k 2 = y(1 + a) = xk implies 3k = (x k)(1 + a). From the equation for x above, we find
3k = ( a(b+a)
1+a k)(1 + a) = a(b + k) k(1 + a), so b + k =
4k+ka
a . Using this in the last equation and
reducing, we get
(1 k 2 ) + (b + k)2 = (1 + a)2
4k + ka 2
(1 k 2 ) + ( ) = (1 + a)2
a
a2 (1 k 2 ) + (4k + ka)2 = a2 (1 + a)2
a2 (ka)2 + (4k)2 + 8k 2 a + (ka)2 = a2 + 2a3 + a4
a4 + 2a3 8k 2 a 16k 2 = 0.
We let a = 2y to obtain
h4 + h3 k 2 h k 2 = 0.
We find h = k 2/3 , hence a = 2k 2/3 . From b = 4k+ka
a k, we find b = 2k 1/3 . Therefore, we can construct
1/3 2/3
2k and 2k using Conways construction.
Exercise 13.3.4.
Let p(x) = x3 + x2 2x 1 and = 2 cos(2/7). By the Rational Root Theorem, if p has a root in Q,
it must be 1 since it must divide its constant term. But p(1) = 1 and p(1) = 1, so p is irreducible
over Q. Therefore, is of degree 3 over Q, hence [Q() : Q] cannot be a power of 2. Since we cant
construct , the regular 7-gon is not constructible by straightedge and compass.
Exercise 13.3.5.
Let p(x) = x2 + x 1 = 0 and = 2 cos(2/5). By the Rational Root Theorem, if p has a root in Q,
it must be 1. Since p(1) = 1 and p(1) = 1, p is irreducible over Q. Hence, is of degree 2 over
Q, so it is constructible. We can bisectpan angle by straightedge and compass, so = cos(2/5) is also
constructible. Finally, as sin(2/5) = 1 cos2 (2/5), sin(2/5) is also constructible. Therefore, the
regular 5-gon is constructible by straightedge and compass.

10
13.4 Splitting Fields and Algebraic Closures

13.4 Splitting Fields and Algebraic Closures


Exercise 13.4.1.

Let f(x) = x4 2. The roots of f are 4 2, 4 2, i4 2 and i 4 2. Hence, the field of f is
splitting
2). So, the splitting field of f has degree [Q(i, 4 2) : Q] = [Q(i, 4 2)
Q(i, 4 : Q( 4 2)][Q( 4 2) : Q] over Q.
Since 4 2 is a root of the irreducible polynomial x4 2 over Q, then [Q( 4 2) : Q] = 4. Furthermore,

since i 6 Q( 4 2),then x2 + 1 is irreducible over Q( 4 2) having i as a root, so [Q(i, 4 2) : Q( 4 2)] = 2.
Therefore, [Q(i, 4 2) : Q] = 8.
Exercise 13.4.2.
Let f (x) = x4 + 2. Let K be the splitting field of f and let L be the splitting field of x4 2, thatis,

L = Q(i, 4 2) (last exercise). We claim K = L, so that [K : Q] = 8 by last exercise. Let = 22 + i 22 .
First we prove L and K, then we prove K = L.
We prove L. This is easy. Let = 4
2. Since L, then 2 = 2 L. We also have i L, so
2, i L implies L.
We prove K. We have to prove i K and 2 K. Let be a root of x4 + 2, so that 4 = 2.
Let be a root of x4 1, so that 4 = 1. Then ()4 = 4 4 = 2, hence is also a root of x4 + 2.
Since the roots of x4 1 are 1, i, the roots of x4 + 2 are and i. Since K is generated over
Q by there roots, then i/ = i K. Now let = 2 K. Since 2 = 4 = 2, then is a root of
2 2
x + 2. Since the roots of x + 2 are i 2 and i 2, then is one of this roots. In either case /i K,
which implies 2 K. Therefore, K.
Now we prove L = K proving both inclusions.
Let be a root of x4 + 2 and be a root of x4 2. Then 4 = 2 and 4 = 2. Note that 2 = i,
so 4 = 1. Hence, ()4 = 4 4 = 2, so is a root of x4 + 2. Then, as we proved earlier, the roots
of x4 + 2 are and i. We also have ()4 = 4 4 = 2, so is a root of x4 2. Then, by last
exercise, the roots of x4 2 are and i. Now, since and are in K, we have K. We also
have i K, so all roots of x4 2 are in K. Since L is generated by these roots, then L K. Similarly,
and are in L, so L; since i L, then all roots of x4 + 2 are in L. Since K is generated by these
roots, we have K L. Therefore, K = L, so [K : Q] = 8.
Exercise 13.4.3.

Let f (x) = x4 + x2 + 1. Note that f (x) = (x2 + x + 1)(x2 x + 1), so the roots of f are 21 i 23 .

Let w = 12 i 23 , so this roots are w, w, w, w, where w denotes the complex conjugate of w (i.e.,

w = 21 + i 23 ). Hence, the splitting field of f is Q(w, w). Since w + w = 1, then Q(w, w) = Q(w).
Furthermore, w is a root of x2 x + 1, that is irreducible over Q since w 6 Q. Therefore, the degree of
the splitting field of f is [Q(w) : Q] = 2.
Exercise 13.4.4.

Let f (x) = x6 4. Note that f (x) = (x3 2)(x3 + 2). The roots of x3 2 are 3 2, 3 2 and 2 3
2,
where is the primitive 3rd root of unity, i.e., = exp(2i/3) = cos(2/3) + i sin(2/3) = 12 + 23 .

Furthermore,
the rootsof x3 + 2 are 3
2, 3
2 and
2 3
2. Therefore, the splitting
field of f is
Q(, 3 2). Then [Q(, 3 2) : Q] = [Q(, 3
2) : Q( 3 2)][Q( 3 2) : Q]. We have that 3 2 is a root of the
irreducible polynomial
x3 2 over Q, so 3 2 has degree
3 over Q. Furthermore, is a root of x2 + x + 1,
3 3
irreducible
over Q( 2), so has degree 2 over Q( 2). Hence, the degree of the splitting field of f is
[Q(, 3 2) : Q] = 6.
Exercise 13.4.5.
We follow the hint. First suppose that K is a splitting field over F . Hence, there exists f (x) F [x]
such that K is the splitting field of f . Let g(x) be an irreducible polynomial in F [x] with a root K.
Let be any root of g. We prove K, so that g splits completely in K[x]. By Theorem 8, there

is an isomorphism : F () F () such that () = . Furthermore, K() is the splitting field for
f over F (a), and K() is the splitting field for f over F (). Therefore, by Theorem 28, extends to

an isomorphism : K() K(). Since K = K(), then [K : F ] = [K() : F ] = [K() : F ], so
K = K(). Thus, K.
Now suppose that every irreducible polynomial in F [x] that has a root in K splits completely in
K[x]. Since [K : F ] is finite, then K = F (1 , . . . , n ) for some 1 , . . . , n . For every 1 i n, let pi
be the minimal polynomial of i over F , and let f = p1 p2 pn . Since every i is in K, every pi has a

11
13.5 Separable and Inseparable Extension

root in K, hence splits completely in K. Therefore, f splits completely in K and K is generated over
F by its roots, so K is the splitting field of f (x) F [x].
Exercise 13.4.6.
(a) Let K1 be the splitting field of f1 (x) F [x] over F and K2 the splitting field of f2 (x) F [x] over
F . Thus, K1 is generated over F by the roots of f1 , and K2 is generated over F by the roots of f2 .
Therefore, f1 f2 splits completely in K1 K2 and K1 K2 is generated over F by its roots, hence is the
splitting field of f1 f2 (x) F [x].
(b) We follow the hint. By last exercise, we have to prove that every irreducible polynomial in F [x] that
has a root in K1 K2 splits completely in (K1 K2 )[x]. So, let f (x) be an irreducible polynomial in
F [x] that has a root, say , in K1 K2 . By last exercise, f splits completely in K1 and splits completely
in K2 . Since K1 and K2 are contained in K, by the uniqueness of the factorization of f in K, the roots
of f in K1 must coincide with its roots in K2 . Hence, f splits completely in (K1 K2 )[x].

13.5 Separable and Inseparable Extension


Exercise 13.5.1.
Let f (x) = an xn + + a1 x + a0 and g(x) = bm xm + + b1 x + b0 be two polynomials. Suppose,
without any loss of generality, that n m. Thus, we can write g(x) = bn xn + + b1 x + b0 , where some
of the last coefficients bi could be zero. We have f (x) + g(x) = (an + bn )xn + + (a1 + b1 )x + (a0 + b0 ),
so

Dx (f (x) + g(x)) = n(an + bn )xn1 + + 2(a2 + b2 )x + (a1 + b1 ) = Dx (f (x)) + Dx (g(x)).


Pn
Now we prove the formula for the product. Let cn = k=0 ak bnk , so that
n
X n
X 2n X
X l 2n
X
f (x)g(x) = ( ak xk )( bk x k ) = ( ak blk )xl = cl xl .
k=0 k=0 l=0 k=0 l=0

Hence,
X2n 2n1
X
Dx (f (x)g(x)) = Dx ( cl xl ) = (l + 1)cl+1 xl ,
l=0 l=0

so the coefficient of xl in Dx (f (x)g(x)) is (l + 1)cl+1 .


Now, we have Dx (f (x)) = nan xn1 + + 2a2 x + a1 , and Dx (g(x)) = nbn xn1 + + 2b2 x + b1 .
So (recall product of polynomials, page 295 of the book)
n
X n
X
Dx (f (x))g(x) = ( kak xk1 )( bk xk )
k=1 k=0
n1
X n
X 2n1
X l
X
=( (k + 1)ak+1 xk )( bk x k ) = ( (k + 1)ak+1 blk )xl ,
k=0 k=0 l=0 k=0

and
n
X n
X
f (x)Dx (g(x)) = ( ak xk )( kbk xk1 )
k=0 k=1
n
X n1
X 2n1
X l
X
k k
=( ak x )( (k + 1)bk+1 x ) = ( ak (l k + 1)blk+1 )xl .
k=0 k=0 l=0 k=0

12
13.5 Separable and Inseparable Extension

Therefore, the coefficient of xl in Dx (f (x))g(x) + Dx (g(x))f (x) is


l
X l
X
( (k + 1)ak+1 blk ) + ( (l k + 1)ak blk+1 )
k=0 k=0
l1
X l
X
= (l + 1)al+1 b0 + ( (k + 1)ak+1 blk ) + ( (l k + 1)ak blk+1 ) + (l + 1)a0 bl+1
k=0 k=1
l
X l
X
= (l + 1)al+1 b0 + ( kak blk+1 ) + ( (l k + 1)ak blk+1 ) + (l + 1)a0 bl+1
k=1 k=1
l
X
= (l + 1)al+1 b0 + ( (l + 1)ak blk+1 ) + (l + 1)a0 bl+1
k=1
l+1
X
= (l + 1)( ak blk+1 ) = (l + 1)cl+1 .
k=0

Since all their coefficients are equal, we conclude Dx (f (x)g(x)) = Dx (f (x))g(x) + Dx (g(x))f (x).
Exercise 13.5.2.
The polynomials x and x + 1 are the only (non-constant, i.e. 6= 0, 1) polynomials of degree 1 over F2 ;
they are clearly irreducible. A polynomial f (x) F2 [x] of degree 2 is irreducible over F2 if and only
if it does not have a root in F2 , that is, exactly when f (0) = f (1) = 1. Hence, the only irreducible
polynomial of degree 2 over F2 is x2 + x + 1. Now, for a polynomial f (x) F2 [x] of degree 4 to be
irreducible, it must have no linear or quadratic factors. We can also apply the condition f (1) = f (0) = 1
to discard the ones with linear factors. Furthermore, f must have an odd number of terms (or it will
be 0), and must have constant term 1 (or x will be a factor). We are left with

x4 + x3 + x2 + x + 1 x4 + x3 + 1
x4 + x2 + 1 x4 + x + 1.

For any of this polynomials to be irreducible, it cant be factorized as two quadratic irreducible factors.
Since x2 + x + 1 is the only irreducible polynomial of degree 2 over F2 , only (x2 + x + 1)2 = x4 + x2 + 1 of
this four is not irreducible. Hence, the irreducible polynomials of degree 4 over F2 are x4 +x3 +x2 +x+1,
x4 + x3 + 1 and x4 + x + 1.
Now, since x + 1 = x 1 in F2 , we have (x + 1)(x4 + x3 + x2 + x + 1) = x5 1. We also calculate
(x + x + 1)(x4 + x + 1)(x4 + x3 + 1) = x10 + x5 + 1. So, the product of all this irreducible polynomials
2

is
x(x + 1)(x2 + x + 1)(x4 + x + 1)(x4 + x3 + 1)(x4 + x3 + x2 + x + 1)
= x(x5 1)(x10 + x5 + 1) = x16 x.

Exercise 13.5.3.
We follow the hint. Suppose d divides n, so that n = qd for some q Z. Then xn 1 = xqd 1 =
(xd 1)(xqdd + xqd2d + . . . + xd + 1). So xd 1 divides xn 1.
Conversely, suppose d does not divide n. Then n = qd + r for some q, r Z with 0 < r < d. Thus
xn 1 = (xqd+r xr )+(xr 1) = xr (xqd 1)+(xr 1) = xr (xd 1)(xqdd +xqd2d +. . .+xd +1)+(xr 1).
Since xd 1 divides the first term, but doesnt divide xr 1 (as r < d), then xd 1 does not divide
xn 1.
Exercise 13.5.4.
The first assertion is analogous to the last exercise.
d
Now, Fpd is defined as the field whose pd elements are the roots of xp x over Fp . Similarly is
defined Fpn . Take a = p. So, d divides n if and only if pd 1 divides pn 1, and that occurs exactly when
d n d d
xp 1 1 divides xp 1 1 (by last exercise). Thus, if d divides n, any root of xp x = x(xp 1 1)
n n d
must be a root of xp x = x(xp 1 1), hence Fpd Fpn . Conversely, if Fpd Fpn , then xp 1 1
n
divides xp 1 1, so d divides n.

13
13.5 Separable and Inseparable Extension

Exercise 13.5.5.
Let f (x) = xp x+a. Let be a root of f (x). First we prove f is separable. Since (+1)p (+1)+a =
p + 1 1 + a = 0, then + 1 is also a root of f (x). This gives p distinct roots of f (x) given by
+ k with k Fp , so f is separable.
Now we prove f is irreducible. Let f = f1 f2 fn where fi (x) Fp [x] is irreducible for all 1 i n.
Let 1 i < j n and let i be a root of fi and j be a root of fj , so that fi is the minimal polynomial
of i and fj the minimal polynomial of j . We prove deg fi = deg fj . Since i is a root of fi , it is a
root of f , hence there exists k1 Fp such that i = + k1 . Similarly, there exists k2 Fp such that
j = + k2 . Thus, i = j + k1 k2 , so fi (x + k1 k2 ) is irreducible having j as a root, so it must
be its minimal polynomial. Hence, fi (x + k1 k2 ) = fj (x), so deg fi = deg fj , as claimed. Since i and
j were arbitrary, then all fi are of the same degree, say q. Then p = deg f = nq, so n = 1 or n = p (as
p is prime). If n = p, then all roots of f are in Fp , so Fp and thus 0 = p + a = a, contrary to
the hypothesis. Therefore, n = 1, so f is irreducible.
Exercise 13.5.6.
n n
By definition, Fpn is the field whose pn elements are the roots of xp x over Fp . Since xp 1 =
n
x(xp 1 1), clearly
n Y
xp 1 1 = (x ).
F
pn

Set x = 0. Then Y n Y
1
1 = () = (1)p
F
pn
F
pn
n n n Y
1 1 1
(1)p (1) = (1)p (1)p
F
pn
n Y
(1)p = .
F
pn

Hence, the product of the nonzero elements is +1 if p = 2 and 1 is p is odd. For p odd and n = 1, we
have Y
1 = ,
F
p

so taking module p we find [1][2] [p 1] = [1], i.e., (p 1)! 1 (mod p).


Exercise 13.5.7.
Let a K such that a 6= bp for every b K. Let f (x) = xp a. We prove that f is irreducible and
inseparable. If is a root of xp a, then xp a = (x )p , so is a multiple root of f (with multiplicity
p), hence f is inseparable. Now, let g(x) be an irreducible factor of f (x). Note that 6 K, otherwise
a = p , contrary to the hypothesis. Then g(x) = (x )k for some k p. Using the binomial theorem,
we have
g(x) = (x )k = xk kxk1 + + ()k .
Therefore, k K. Since 6 K, then k = p, so g = f . Hence, f is irreducible. We conclude that K()
is an inseparable finite extension of K.
Exercise 13.5.8.
Let f (x) = an xn + . . . + a1 x + a0 Fp [x]. Since Fp has characteristic p, then (a + b)p = ap + bp for any
a, b Fp . Easily we can generalize this to a finite number of terms, so that (x1 + +xn )p = xp1 + +xpn
for any x1 , , xn Fp . Furthermore, by Fermats Little Theorem, ap = a for every a Fp . So, over
Fp , we have

f (x)p = (an xn + . . . + a1 x + a0 )p = apn xnp + . . . + ap1 xp + ap0 = an xnp + . . . + a1 xp + a0 = f (xp ).

14
13.6 Cyclotomic Polynomials and Extensions

Exercise 13.5.9.
By the binomial theorem, we have
pn  
pn
X pn
(1 + x) = xi ,
i=0
i

so the coefficient of x in the expansion of (1 + x) is pn


pi

pn
pi .
Since Fp has characteristic p, we have (1+x)pn = 1+xpn = (1+xp )n , so over Fp this is the coefficient
of (xp )i in (1 + xp )n . Furthermore, (1 + x)pn = (1 + xp )n implies
n   pn  
X n
p n p i
X pn i
(1 + x ) = (x ) = x = (1 + x)pn
i=0
i i=0
k
pn n
 
over Fp , hence pi i (mod p).
Exercise 13.5.10.
This is equivalent to prove that for any prime number p, we have f (x1 , x2 , . . . , xn )p = f (xp1 , xp2 , . . . , xpn )
in Fp [x1 , x2 , . . . , xn ]. Let
X
f (x1 , x2 , . . . , xn ) = a1 ,...,n x11 . . . xnn
1 ,...,n =0

be an element of Fp [x1 , x2 , . . . , xn ].
Since Fp has characteristic p, then (x1 + + xn )p = xp1 + + xpn for any x1 , , xn Fp .
Furthermore, by Fermats Little Theorem, ap = a for every a Fp . Hence, over Fp we have
X X
f (x1 , x2 , . . . , xn )p = ( a1 ,...,n x11 . . . xnn )p = (a1 ,...,n x11 . . . xnn )p
X X
= ap1 ,...,n (x11 . . . xnn )p = a1 ,...,n (xp 1 pn p p p
1 . . . xn ) = f (x1 , x2 , . . . , xn ).

Exercise 13.5.11.
Let f (x) F [x] with no repeated irreducible factors in F [x]. We can suppose f is monic. Then
f = f1 f2 fn for some monic irreducible polynomials fi (x) F [x]. Since F is perfect, f is separable,
hence all fi has distinct roots. Thus, f splits in linear factors in the closure of F , hence splits in linear
factors in the closure of K. Therefore, f (x) has no repeated irreducible factors in K[x].

13.6 Cyclotomic Polynomials and Extensions


Exercise 13.6.1.
Since (m n )mn = 1, then m n is an mnth root of unity. Now, let 1 k < mn. Then (m n )k = m k k
n .
k k k
For a contradiction, suppose m n = 1. Then, m = 1 and nk = 1, hence m divides k and n divides k,
so mn divides k (as m 6= n). Since 1 k < mn, this is impossible. So, (m n )k 6= 1 for all 1 k < mn
and thus the order of m n is mn. Therefore, m n generates the cyclic group of all mnth roots of unity,
that is, m n is a primitive mnth root of unity.
Exercise 13.6.2.
Since (nd )(n/d) = nn = 1, then nd is an (n/d)th root of unity. Now let 1 k < (n/d). Then (nd )k = nkd .
Since 1 kd < n, then nkd 6= 1, so (nd )k 6= 1. Hence, the order of nd is (n/d), so it generates the cyclic
group of all (n/d)th roots of unity, that is, nd is a primitive (n/d)th root of unity.
Exercise 13.6.3.
Let F be a field that contains the nth roots of unity for n odd and let be a 2nth root of unity. If
n = 1, then F , so suppose n 6= 1. Since 2n = 1, then n is a root of x2 1. Since the roots of
this polynomial are 1 and 1, and n 6= 1, then n = 1. Hence, ()n = (1)n ()n = (1)n+1 = 1
(since n is odd), so F . Since F is a field, then F .
Exercise 13.6.4.
k k
Let F be a field with char F = p. The roots of unity over F are the roots of xn 1 = xp m 1 = (xm 1)p ,
so are the roots of xm 1. Now, since m is relatively prime to p, so is xm 1 and its derivative mxm1 ,

15
13.6 Cyclotomic Polynomials and Extensions

so xm 1 has no multiple roots. Hence, the m different roots of xm 1 are precisely the m distinct nth
roots of unity over F .
Exercise 13.6.5.

We use the inequality (n) n/2 for all n 1. Let K be an extension of Q with infinitely many
2
roots of unity. Let N N. Then, there exits n N such that n > 4N and there exits some nth root
of unity K. Then [K : Q] [Q() : Q] = (n) n/2 > N . Since N was arbitrary, we conclude
that [K : Q] > N for all N N, so [K : Q] is infinite. Therefore, in any finite extension of Q there are
only a finite number of roots of unity.
Exercise 13.6.6.
Since 2n (x) and n (x) are irreducible, they are the minimal polynomial of any of its roots. Thus, it
is sufficient to find a common root for both. Let 2 be the primitive 2th root of unity and let n be a
primitive nth root of unity. Note that 2 = 1, so that 2 n = n . Since n is odd, 2 and n are relatively
prime. So, by Exercise 1, 2 n is a primitive 2nth root of unity, i.e, a root of 2n (x). Furthermore,
n is clearly a root of n (x). Thus, n is a common root for both 2n (x) and n (x), hence
2n (x) = n (x).
Exercise 13.6.7.
The
P Mbius Inversion Formula sates that if f (n) is defined for all nonnegative integers and F (n) =
( nd ). So lets start with the formula
P
d|n f (d), then f (n) = d|n (d)F
Y
xn 1 = d (x).
d|n

We take natural logarithm in both sides and obtain


Y X
ln(xn 1) = ln( d (x)) = ln d (x).
d|n d|n

So, we use the Mbius Inversion Formula for f (n) = ln n (x) and F (n) = ln(xn 1) to obtain
X X
ln n (x) = (d) ln(xn/d 1) = ln(xn/d 1)(d) .
d|n d|n

Hence, taking exponentials we obtain


X Y Y
n (x) = exp( ln(xn/d 1)(d) ) = (xn/d 1)(d) = (xd 1)(n/d) .
d|n d|n d|n

Exercise 13.6.8.
x`1 (x1)`
(a) Since p is prime, in Fp [x] we have (x 1)p = xp 1, so ` (x) = x1 = x1 = (x 1)l1 .
(b) Note that has order ` as being a primitive `th root of unity. Since pf 1 mod `, then pf 1 = q`
f
for some integer q, hence p 1 = q` = 1, so Fpf . Now we prove that f is the smallest integer
n
with that property. Suppose Fpn for some n. Then is a root of xp 1 1, hence ` divides pn 1
(see Exercise 13.5.3). Since f is the smallest power of p such that pf 1 mod `, is the smallest integer
such that ` divides pf 1, so n l, as desired. This in fact proves that Fp () = Fpf , so the minimal
polynomial of over Fp has degree f .
(c) Since a Fp (), clearly Fp ( a ) Fp (). For the other direction we follow the hint. Let b the
the multiplicative inverse of a mod `, i.e ab 1 mod `. Then ( a )b = , so Fp ( a ) and thus
Fp () Fp ( a ). The equality follows.
Now, consider ` (x) as a polynomial over Fp [x]. Let i for 1 i ` be ` distinct primitive `th roots
of unity. The minimal polynomial of each i has degree f by part (b). Hence, the irreducible factors of
` (x) have degree f . Since ` have degree ` 1, then there must be `1 f factors, and all of them are
different since ` (x) is separable.
(d) If p = 7, then 7 (x) = (x 1)6 by part (a). If p 1 mod 7, then f = 1 in (b) and all roots have
degree 1, so 7 (x) splits in distinct linear factors. If p 6 mod 7, then f = 2 is the smallest integer

16
13.6 Cyclotomic Polynomials and Extensions

such that pf = p2 36 1 mod 7, so we have 3 irreducible quadratics. If p 2, 4 mod 7, then f = 3 is


the smallest integer such that p3 23 , 43 8, 64 1 mod 7, so we have 2 irreducible cubics. Finally, if
p 3, 5 mod 7, then f = 6 is the smallest integer such that p6 36 , 56 729, 15626 1 mod 7, hence
we have an irreducible factor of degree 6.
Exercise 13.6.9.
Let A be an n by n matrix over C for which Ak = I for some integer k 1. Then the minimal polynomial
of A divides xk 1. Since we are working over C, there are k distinct roots of this polynomial, so the
minimal polynomial ofA canbe split in linear factors. Hence, A is diagonalizable.
1
Now consider A = where is an element of a field of characteristic p.
0 1  
n 1 n
Computing powers of A, we can prove (by induction) that A = for every positive integer
0 1
p
n. Since p = 0, then A = I. Now, if A is diagonalizable, there exists some non-singular matrix P
such that A = P DP 1 , where D is a diagonal matrix whose diagonal entries are the eigenvalues of A.
Since A has only one eigenvalue 1, then D = I, and thus A = P IP 1 = I. So, if A is diagonalizable, it
must be = 0.
Exercise 13.6.10.
Let a, b Fpn . Then (a + b) = (a + b)p = ap + bp = (a) + (b), and (ab) = (ab)p = ap bp = (a)(b),
so is and homomorphism. Moreover, if (a) = 0, then ap = 0 implies a = 0. Hence, is injective.
Since Fpn is finite, then is also surjective so it is an isomorphism. Furthermore, since every element
n n
of Fpn is a root of xp x, then n (a) = ap = a for all a Fpn , so n is the identity map. Now, let m
m
be an integer such that m is the identity map. Then ap = a for all a Fpn , so every element of Fpn
m
must be a root of xp x. Hence, Fpn Fpm and thus n divides m (Exercise 13.5.4), so n m.
Exercise 13.6.11.
Note that the minimal polynomial of is xn 1, for if satisfies some polynomial xn1 + + a1 x + a0
n1
of degree n 1 (or less) with coefficients in Fp , then xp + + a1 xp + a0 for all x Fpn , which
is impossible. Since Fpn has degree n as a vector space over Fp , then xn 1 is also the characteristic
polynomial of , hence is the only invariant factor. Therefore, the rational canonical form of over Fp
is the companion matrix of xn 1, which is

0 0 0 1
1 0 0 0

0 1 0 0
.
.. .. . . . .
. . . .. ..
0 0 1 0

Exercise 13.6.12.
Well work over the algebraic closure of Fpn , to ensure the field to contain all eigenvalues. In last exercise
we proved that the minimal and characteristic polynomial of is xn 1. Moreover, the eigenvalues of
are the nth roots of unity. We use Exercise 4 and write n = pk m for some prime p and some m relatively
k
prime to p, so that xn 1 = (xm 1)p and we get exactly m distinct nth roots of unity, each one with
k
multiplicity p . Since all the eigenvalues are zeros of both the minimal and characteristic polynomial of
multiplicity pk , we get m Jordan blocks of size pk . Now, fix a primitive mth root of unity, say . Then,
each Jordan block each of the form
i
1 0 0 0
0 i 1 0 0
0 0 i 0 0

Ji = .

.. .. . . .. ..
.. . . . . .

0 0 0 i 1
0 0 0 0 i

17
13.6 Cyclotomic Polynomials and Extensions

for some 0 i m 1. We already know the Jordan canonical form is given by



J0 0 0
0 J1 0

0
0 0 .
.. .. . . ..
. . . .
0 0 Jm1

Exercise 13.6.13.
(a) Z is a division subring of D, and it is commutative by definition of the center, so Z is a field. Since
it is finite, its prime subfield is Fp for some prime p, so it is isomorphic to Fpm for some integer m. Let
q = pn , so that Z is isomorphic to Fq . Since D is a vector space over Z, then |D| = q n for some integer
n.
(b) Let x D and let CD (x) be the set of the elements in D that commutes with x. Clearly Z CD (x).
We prove that every element a CD (x) has an inverse in CD (x). Since a CD (x), then ax = xa.
Since D is a division ring, then a1 D. Moreover, we have a1 ax = a1 xa and thus x = a1 xa, so
xa1 = a1 x and a1 CD (x). Hence, CD (x) is a division ring. Now, since Z CD (x), then CD (x)
is a Z-vector space, so |CD (x)| = q m for some integer m. If x 6 Z, then CD (x) is a proper subset of D
and hence m < n.
(c) The class equation for the group D is
r
X
|D | = |Z(D )| + |D : CD (xi )|,
i=1

where the xi are the representatives of the distinct conjugacy class. By (a) we have |D | = q n 1,
qn 1 qn 1
|Z(D )| = q 1 and |CD (xi )| = q mi 1. Then |D : CD (xi )| = = mi . Replacing in
|CD (xi )| q 1
the class equation we obtain
r r
X qn 1 X qn 1
q n 1 = (q 1) + = (q 1) + .
i=1
|CD (xi )| i=1
q mi 1

qn 1
(d) Since |D : CD (xi )| is an integer, then |D : CD (xi )| = is an integer. Hence, q mi 1
q mi 1
divides q n 1, so (Exercise 13.5.4) mi divides n. Since mi < n (no xi is in Z), no mth i root of unity is a
nth root of unity. Therefore, as n (x) divides xn 1, it must divide (xn 1)/(xmi 1) for i = 1, 2, . . . , r..
Letting x = q we have n (q) divides (q n 1)/(q mi 1) for i = 1, 2, . . . , r.
(e) From (d), n (q) divides (q n 1)/(q mi 1) for i = 1, 2, . . . , r, so the class equation in (c) implies
n (q) divides q 1. Now, let 6= 1 be a nth root
Q of unity. In the complex plane q is closer to 1 that
is, so |q | > |q 1| = q 1. Since n (q) = primitive (q ) divides q 1, this is impossible unless
n = 1. Hence, D = Z and D is a field.
Exercise 13.6.14.
We follow the hint. Let P (x) = xn + + a1 x + a0 be a monic polynomial of degree 1 over Z. For
a contradiction, suppose there are only finitely many primes dividing the values P (n), n = 1, 2, . . ., say
p1 , p2 , . . . , pk . Let N be an integer such that P (N ) = a 6= 0. Let Q(x) = a1 P (N + ap1 p2 . . . pk x).
Then, using the binomial theorem, we have

Q(x) = a1 P (N + ap1 p2 . . . pk x)
= a1 ((N + ap1 p2 . . . pk x)n + + a1 (N + ap1 p2 . . . pk x) + a0 )
= a1 (N n + an1 N n1 + + a1 N + a0 + R(x))
= a1 (P (N ) + R(x))
= 1 + a1 R(x)

18
13.6 Cyclotomic Polynomials and Extensions

for some polynomial R(x) Z[x] divisible by ap1 p2 . . . pk . Hence, Q(x) Z[x]. Moreover, for all n Z+ ,
P (N + ap1 p2 . . . pk n) a (mod p1 , p2 , . . . , pk ), so Q(n) = a1 P (N + ap1 p2 . . . pk n) a1 a = 1 (mod
p1 , p2 , . . . , pk ). Now let m be a positive integer such that |Q(m)| > 1, so that Q(m) 1 (mod pi ) for
all i. Therefore, none of the pi s divide Q(m). Since |Q(m)| > 1, there exists a prime q 6= pi for all i
such that q divides Q(m). Then q divides aQ(m) = P (N + ap1 p2 . . . pk m), contradicting the fact that
only the primes p1 , p2 , . . . , pk divides the numbers P (1), P (2), . . ..
Exercise 13.6.15.
We follow the hint. Since m (a) 0 (mod p), then am 1 (mod p). Hence, there exist b such that
ba 1 mod p (indeed, b = am1 ), so a is relatively prime to p. We prove that the order of a is m.
For a contradiction, suppose ad 1 (mod p) for some d dividing m, so that d (a) 0 (mod p) for
some d < m. Thus, a is a multiple root of xm 1, so is also a root of its derivative mam1 . Hence,
mam1 0 mod p, impossible since p does not divide m nor a. Therefore, the order of a in (Z/pZ) is
precisely m
Exercise 13.6.16.
Let p be an odd prime dividing m (a). If p does not divide m, then, by (c), a is relatively prime to p
and the order of a in F
p is m. Since |Fp | = p 1, this implies m divides p 1, that is, p 1 (mod m).

Exercise 13.6.17.
By Exercise 14, there are infinitely many primes dividing m (1), m (2), m (3), . . .. Since only finitely
of them can divide m, then, by Exercise 16, there must exists infinitely many primes p with p 1 (mod
m).

Please send comments, suggestions and corrections by e-mail, or at website.


https://positron0802.wordpress.com
positron0802@mail.com

19

You might also like